Your-Doctor
Multiple Choice Questions (MCQ)


Quiz Categories Click to expand

Category: Cardiology--->Valvular heart disease and Endocarditis
Page: 2

Question 6# Print Question

A 68-year-old man is referred for assessment of an ejection systolic murmur after presenting with worsening breathlessness. Image loops taken from his transthoracic echocardiogram are shown 

The following measurements were obtained during transthoracic echo:

  • mean gradient across aortic valve, 30 mmHg
  • aortic valve area (by continuity equation), 1.0 cm2

Coronary angiography demonstrated mild atheroma without any significant disease.

Which one of the following would be the most useful next investigation?

A. Repeat transthoracic echocardiogram in 6 months
B. Transoesophageal echocardiography
C. Dobutamine stress echocardiography
D. Exercise tolerance test for risk stratification
E. Repeat transthoracic echocardiogram in 12 months


Question 7# Print Question

A 79-year-old retired farmer with known aortic stenosis (AS) returns for his annual surveillance echocardiogram. He remains physically active with no symptoms. His BP is 180/110 mmHg.

The following summary is obtained:

  • Severe AS—peak velocity has increased from 3.8 m/s a year ago to 4.0 m/s today
  • Mean gradient, 40 mmHg
  • Aortic valve area, 1.0 cm2
  • Mild LVH
  • Good overall LV systolic function

Which one of the following statements is correct?

A. A statin should be prescribed to reduce the rate of AS progression
B. An antihypertensive drug should be prescribed
C. The increase in peak velocity of ≥0.2 m/s/year suggests that surgery should be considered
D. An exercise tolerance test (ETT) is unsafe in asymptomatic severe AS
E. An elevated BNP of 120 pg/ml suggests that surgery should be considered


Question 8# Print Question

Past Medical History (An 82-year-old retired solicitor presents to the ED with chest pain radiating to his jaw. He has hypertension treated with ramipril 5 mg bd but is otherwise normally fit and well. His admission ECG shows atrial fibrillation with a ventricular rate of 90 bpm, LVH, and widespread ST segment depression. His peak troponin is 110 ng/L (normal <30 ng/L). He is started on treatment for an acute coronary syndrome and listed for an inpatient angiogram. You are asked to perform a bedside echocardiogram as a systolic murmur is heard on the post-take ward round. 

coronary angiogram showed:

  • LMS: mild atheroma
  • LAD: severe (90%) proximal stenosis; good distal target
  • LCx: small vessel with diffuse distal atheroma
  • RCA: dominant; moderate (50–60%) mid-vessel focal stenosis.)

Three years later, is admitted to hospital with chest pain. A repeat echocardiogram shows a heavily calcified aortic valve with a peak velocity of 4.8 m/s, valve area of 0.8 cm2 , and moderately impaired LV systolic function. Two years ago he had a right upper lobe lung lobectomy with chemoradiotherapy for a localized primary bronchogenic carcinoma. Your consultant has asked you to write a referral letter to the ‘heart team’ at the regional tertiary centre to consider this patient for a transcatheter aortic valve implantation (TAVI).

Which one of the following is a contraindication for TAVI?

A. Plaques with mobile thrombi in the ascending aorta or arch
B. Porcelain aorta
C. Home oxygen therapy
D. Severe peripheral vascular disease
E. LVEF <30%


Question 9# Print Question

The Doppler profile shown below is taken from a patient presenting with breathlessness.

What is this Doppler profile least likely to be compatible with?

A. Chronic secondary mitral regurgitation
B. Severe mitral stenosis
C. Aortic stenosis with concentric LVH
D. Cardiac amyloidosis
E. Diasystolic dysfuntion


Question 10# Print Question

You are asked to review an echocardiogram of a 82-year-old woman who has both severe aortic stenosis (AS) and severe mitral regurgitation (MR). All the following statements are true in patients with combined or multiple valve lesions except:

A. Associated MR may lead to underestimation of the severity of AS
B. Severe AS may lead to overestimation of coexisting MR
C. Significant aortic regurgitation (AR) lengthens the Doppler pressure half-time (PHT) in mitral stenosis (MS)
D. The presence of significant AR may overestimate the gradient across the aortic valve
E. Planimetry should be utilized




Category: Cardiology--->Valvular heart disease and Endocarditis
Page: 2 of 18